Question:
Les équations de Maxwell peuvent-elles être dérivées de la loi de Coulomb et de la relativité spéciale?
user1247
2011-01-22 23:33:05 UTC
view on stackexchange narkive permalink

En tant qu'exercice, je me suis assis et j'ai dérivé le champ magnétique produit par le déplacement de charges pour quelques situations artificielles. J'ai commencé avec la loi de Coulomb et la relativité spéciale. Par exemple, j'ai dérivé le champ magnétique produit par un courant $ I $ dans un fil infini. C'est un effet relativiste; dans le cadre d'une charge de test, la densité électronique augmente ou diminue par rapport à la densité de protons dans le fil en raison d'une contraction de longueur relativiste, en fonction du mouvement de la charge de test. L'effet net est un champ de Coulomb dépendant de la trame dont l'effet sur une charge de test est exactement équivalent à celui d'un champ magnétique selon la loi de Biot – Savart.

Ma question est: Les équations de Maxwell peuvent-elles être dérivées en utilisant uniquement la loi de Coulomb et la relativité spéciale?

Si c'est le cas, et que le champ $ B $ est dans tous les cas un effet purement relativiste, alors les équations de Maxwell peuvent être réécrites sans référence à un champ $ B $. Cela laisse-t-il encore de la place aux monopôles magnétiques?

Je me souviens vaguement, lorsque j'étais au lycée, d'avoir trouvé un livre qui traitait de l'E & M de premier cycle en supposant que la RS est correcte depuis le début et en faisant quelque chose comme ça. Je ne me souviens pas du titre, cependant (ou s'il était bon), mais si vous voulez voir cela en détail, vous pouvez essayer de le chercher?
@Jeremy - Le livre auquel vous pensez est probablement * Electricity and Magnetism * par E. Purcell (qui fait partie de la série Berkeley Physics). Un très bon livre, d'ailleurs.
Oui! Je crois que c'est à cela que je pensais.
Oui, j'ai utilisé ce livre à l'université. Très bon livre. Mais cela ne va pas "jusqu'au bout" et ne dérive pas les équations de Maxwell (si je me souviens bien).
Vous devez également supposer que la charge est une répulsion scalaire et de même charge. Ensuite, la dérivation est contenue dans le livre EM de Purcell.
Oui, c'est possible et cela est réalisé en détail par Haskell dans ce document: http://richardhaskell.com/files/Special%20Relativity%20and%20Maxwells%20Equations.pdf Voir aussi la discussion très éclairante qui suit sur les problèmeslors de l'application des équations de Maxwell à des charges de source accélérées.
Treize réponses:
Luboš Motl
2011-01-22 23:47:33 UTC
view on stackexchange narkive permalink

Les équations de Maxwell découlent des lois de l'électricité combinées aux principes de la relativité restreinte. Mais ce fait n'implique pas que le champ magnétique en un point donné est moins réel que le champ électrique. Bien au contraire, la relativité implique que ces deux champs doivent être également réels.

Lorsque les principes de relativité restreinte sont imposés, le champ électrique $ \ vec {E} $ doit être incorporé dans un objet qui se transforme de manière bien définie sous les transformations de Lorentz - c'est-à-dire lorsque la vitesse de l'observateur est modifiée. Parce qu'il n'existe pas de "force électrique scalaire", et pour d'autres raisons techniques que je ne veux pas expliquer, $ \ vec {E} $ ne peut pas faire partie d'un 4-vecteur dans l'espace-temps, $ V _ {\ mu } $.

Au lieu de cela, il doit s'agir des composantes $ F_ {0i} $ d'un tenseur antisymétrique à deux indices, $$ F _ {\ mu \ nu} = - F _ {\ nu \ mu} $ $ De tels objets, généralement connus sous le nom de tenseurs, savent comment se comporter sous les transformations de Lorentz - lorsque l'espace et le temps sont tournés l'un dans l'autre comme la relativité le rend obligatoire.

Les indices $ \ mu, \ nu $ take valeurs $ 0,1,2,3 $ soit $ t, x, y, z $. En raison de l'antisymétrie ci-dessus, il y a 6 composantes inéquivalentes du tenseur - les valeurs de $ \ mu \ nu $ peuvent être $$ 01,02,03; 23,31,12. $$ Les trois premières combinaisons correspondent aux trois composantes du champ électrique $ \ vec {E} $ alors que les trois dernières combinaisons portent l'information sur le champ magnétique $ \ vec {B} $.

Quand j'avais 10 ans, je pensais aussi que le champ magnétique aurait pu être juste un artefact du champ électrique mais il ne peut pas en être ainsi. Au lieu de cela, les champs électrique et magnétique à chaque point sont complètement indépendants l'un de l'autre. Néanmoins, la symétrie de Lorentz peut les transformer l'un en l'autre et les deux sont nécessaires pour que leur ami puisse se transformer en quelque chose dans un système inertiel différent, de sorte que la symétrie sous le changement du système inertiel ne soit pas perdue.

Si vous ne commencez qu'avec le champ électrique $ E_z $, le composant $ F_ {03} $ est différent de zéro. Cependant, lorsque vous augmentez le système dans la direction $ x $, vous mélangez la coordonnée temporelle $ 0 $ avec la coordonnée spatiale $ x $ $ 1 $. Par conséquent, une partie du champ $ F_ {03} $ est transformée en le composant $ F_ {13} $ qui est interprété comme le champ magnétique $ B_y $, jusqu'à un signe.

Alternativement, on peut décrire l'électricité par le potentiel électrique $ \ phi $. Cependant, la densité d'énergie de la densité de charge $ \ rho = j_0 $ doit être un tenseur avec deux indices temporels, $ T_ {00} $, donc $ \ phi $ lui-même doit également porter un index temporel. Ce doit être que $ \ phi = A_0 $ pour certains $ A $ à 4 vecteurs. L'ensemble de ces 4 vecteurs doit exister par relativité, y compris les composantes spatiales $ \ vec {A} $, et un nouveau champ $ \ vec {B} $ peut être calculé comme la boucle de $ \ vec {A} $ tandis que $ \ vec {E} = - \ nabla \ phi- \ partial \ vec {A} / \ partial t $.

Vous vouliez apparemment prouver l'absence des monopôles magnétiques par prouver l'absence du champ magnétique lui-même. Eh bien, excusez-vous d'avoir interrompu votre plan de recherche: cela ne peut pas fonctionner. Les aimants sont vraiment réels. Et si cela vous intéresse, l'existence de monopôles magnétiques est inévitable dans toute théorie cohérente de la gravité quantique. En particulier, deux pôles d'un aimant en forme d'haltère peuvent s'effondrer en une paire de trous noirs qui posséderont inévitablement les charges monopôles magnétiques (opposées). Les trous noirs les plus légers possibles (masse de Planck) avec des charges monopôles magnétiques seront des «preuves de concept» de particules élémentaires lourdes avec des charges magnétiques - cependant, des particules plus légères avec les mêmes charges peuvent parfois exister.

Il est donc vrai que les équations de Maxwell peuvent être réécrites sans référence à un champ B?
Nan. Il doit toujours y avoir un champ $ B $. Il peut être calculé à partir d'un autre champ, comme le vecteur potentiel $ A $, où $ B = \ mbox {curl} A $, mais $ B $ doit exister et existe. Ses composants doivent être et sont indépendants des composants du champ électrique. Et la relativité implique que $ B $ * existe *, plutôt que qu'il n'existe * pas *. Ma réponse précédente n'était-elle pas claire ou y a-t-il une raison plus profonde pour laquelle vous posez à nouveau la même question?
Votre réponse précédente n'était pas claire en ce qu'elle n'explique pas pourquoi je suis capable (dans quelques exemples artificiels, certes) de résoudre les équations de mouvement sans référence à un champ B. Tout ce que j'ai à faire est de montrer comment le champ E se transforme sous les boosts de Lorentz, et je peux le faire sans introduire de champ B. N'ai-je pas fait ces exemples correctement, ou s'agit-il d'exceptions chanceuses parce qu'elles sont artificielles?
Cher utilisateur1247, oui, en effet, la relativité nous oblige à découvrir comment $ E $ se transforme sous les boosts de Lorentz. Et si vous faites le calcul correctement, et que je l'ai bien fait, vous découvrirez que $ E $ ne peut pas se transformer en lui-même. Il doit se transformer en un autre champ, $ B $. Même si vous commencez avec $ E $ non nul et $ B = 0 $, le boost de Lorentz par vitesse $ v $ créera un nouveau champ $ B = v \ times E $, et il n'y a aucun moyen de l'éviter. Pourriez-vous s'il vous plaît lire ma réponse avant de répéter à nouveau votre idée fausse? Merci.
J'ai lu attentivement votre réponse et vous ne répondez toujours pas à mes questions. Quand je fais mes exemples, nulle part je ne dois postuler un "nouveau champ". Je commence simplement par la loi de Coulomb et SR, et je fais le calcul, et les mathématiques montrent qu'une particule subit des forces qui peuvent être décrites de manière efficace par un «nouveau champ». Ceci est analogue à la force de Coriolis. La gravité associée à un référentiel tournant implique-t-elle un nouveau «champ de Coriolis»? Bien sûr que non, mais cela peut être décrit par un seul.
Je respecte vraiment vos connaissances en physique Lubos, et je vous remercie sincèrement d'avoir pris le temps de répondre à cette question. Je sais que vous _pouvez_ répondre à la question, mais je pense que vous ne prenez pas le temps d'essayer de lire et de comprendre ma question assez attentivement pour le faire.
Vous écrivez: "nulle part je ne dois postuler un" nouveau champ "." - C'est exactement pourquoi je vous dis que vous n'avez pas réussi à lire mon texte parce que mon texte montre qu'il faut * postuler * un nouveau champ pour que la relativité tienne. C'est justement là-dessus, et c'est précisément ce que vous avez demandé. Vous n'aimez tout simplement pas la réponse, n'est-ce pas? Le champ magnétique ne peut pas être «efficace» si le champ électrique est réel. Il doit être exactement tout aussi réel car ils sont liés par une symétrie: ils font partie d'un tenseur.
La force de Coriolis ne découle * pas * de l'électricité ou du magnétisme. Pourquoi mixez-vous ce truc ici? En relativité générale, cependant, on exige que la gravité soit indiscernable de l'accélération. Pour ce faire, on prouve qu'il doit y avoir un nouveau champ qui se souvienne de l'accélération locale. On pourrait l'appeler le champ de Coriolis mais on l'appelle généralement plus généralement, le tenseur métrique. Oui, le tenseur métrique découle également de symétries et de principes. Vous vous trompez donc aussi sur la force de Coriolis. Ces champs peuvent être des manifestations de chaînes - mais ils doivent toujours être réels et indépendants.
La réponse de Lubos est très bonne et très précise. Je suis entièrement abonné. Je ne suis qu'un peu perplexe par le dernier paragraphe de la réponse, où il dit que l'existence de monopôles magnétiques est inévitable dans toute théorie cohérente de la gravité quantique. L'argument avancé est que deux pôles d'un aimant en forme d'haltère peuvent s'effondrer en une paire de trous noirs qui "posséderont inévitablement les charges monopôles magnétiques (opposées)". Si je casse un aimant, je n'obtiens pas deux monopôles, bien sûr. Je reçois deux aimants. Qu'est-ce qui empêcherait les deux trous noirs de faire exactement la même chose, et de se comporter comme deux aimants, sans
... un monopole autour? (c'est le reste du commentaire du Dr Rovelli, qui a été coupé par le système)
Cher @Carlo Rovelli, merci pour vos commentaires. Et oui, bien sûr, un aimant (dipôle) se brise en deux aimants (dipôles). Mais c'est parce que les dipôles magnétiques sont portés par les spins des électrons et que l'on ne peut briser l'aimant (dipôle) que le long des surfaces éloignées des électrons, donc on obtient toujours deux aimants (dipôles) et pas de monopôles. Cependant, les trous noirs ne sont pas «faits d'électrons», de sorte que la disparition de leurs charges monopôles magnétiques n'a pas à tenir, ce qui signifie, selon le principe de Gell-Mann, qu'elle ne tient généralement pas.
Permettez-moi d'ajouter quelques références. Regardez par exemple à http://arxiv.org/abs/hep-th/9404076 qui fait référence à l'article d'Affleck-Manton qui a généralisé l'effet Schwinger aux champs magnétiques. Tout comme le champ électrique de Schwinger produira des paires électron-positon, le champ magnétique produira des paires monopôle-antimonopole. Les conditions permettant cette production sont inévitables en QG où les monopôles peuvent être représentés par des trous noirs chargés de monopôles.
Je me rends compte que jusqu'ici mes commentaires auraient pu sonner circulaires. Mais la production de paires de trous noirs peut être mise en évidence par un instanton gravitationnel-magnétique qui mène aux deux trous noirs chargés magnétiquement monopolaires qui s'éloignent l'un de l'autre. Un tel instanton satisfait toutes les quantifications et propriétés locales que l'on peut imposer dans GR, il est donc physique et implique que la probabilité du processus est non nulle. Toute règle supplémentaire interdisant de tels instantons violerait la localité. L'instanton continue analytiquement une solution Ernst de 1976. Le papier Garfinkle Strominger 1991 est également recommandé.
Lubos, connaissez-vous cette référence aux monopôles magnétiques dans un cristal http://www.sciencedaily.com/releases/2009/09/090903163725.htm? Une question que je me pose pour n'avoir pas encore vu de monopôles big bang: leur rareté pourrait-elle être liée au même mécanisme qui expliquerait le fait que nous n'avons pas non plus d'égalité entre particules et antiparticules macroscopiquement?
@LubošMotl fait votre déclaration «Et si vous êtes intéressé, l'existence de monopôles magnétiques est inévitable dans toute théorie cohérente de la gravité quantique.impliquent que les physiciens qui étudient la gravitation quantique croient que la gravitation quantique existe aussi des monopôles magnétiques?Je n'avais pas entendu cela comme condition / conséquence de l'existence de la gravité quantique.
Cher @Stan,, je suis convaincu que la plupart des experts avec plus de 5000 citations sur les papiers de gravité quantique conviendront avec moi que les monopoles magnétiques sont inévitables en gravité quantique.C'est vraiment parce que vous pouvez créer des trous noirs avec un flux magnétique confiné - imaginez que deux extrémités d'un grand aimant à barreaux s'effondrent séparément dans des trous noirs.Donc ces objets existent sous la forme de trous noirs et même si et quand le trou noir évapore toutes les particules avec $ Q_m = 0 $, quelque chose avec $ Q_m \ neq 0 $ doit quand même être laissé.Les particules élémentaires monopôles magnétiques peuvent être identifiées avec les micro-états BH les plus légers
Alors, pourquoi n'y a-t-il pas d'équivalent d'aimant de gravité dans le monopole d'aimant de gravité ou GR?
Les charges dans l'électromagnétisme et la gravité portent des spins différents - des représentations sous le groupe de Lorentz.Ce sont les quantités conservées, et il se trouve qu'elles sont le Q électrique en tant que générateur scalaire d'un U (1) en électricité, Q_m supplémentaire en tant que générateur du S-dual U (1) en magnétisme.En gravité, les charges conservées sont le vecteur énergie-impulsion à 1 indice.L'électromagnétisme et la gravité ne sont tout simplement pas exactement les mêmes ou isomorphes - ils sont simplement analogues à bien des égards.
L'argument sur la paire de monopôles de l'effondrement d'un aimant est erroné.L'aimant est composé de matière normale et d'électrons.À un moment donné de l'effondrement, l'aimant se fragmentera et fabriquera deux (ou plus) aimants plus petits.A chaque point de l'effondrement, les équations classiques de Maxwell (avec gravité) seront satisfaites et il n'y aura pas de monopôles.La création de monopôles dans ce processus implique une nouvelle physique.L'effondrement gravitationnel d'un aimant n'est donc pas du tout utile pour justifier des monopôles.
Vous vous trompez.En effet, les monopôles sont une nouvelle physique et la création de monopôles prouve en effet une nouvelle physique.Mais l'argument est valide, c'est donc une preuve que la physique au-delà de SM doit exister.Votre affirmation selon laquelle la matière se réorganise toujours pour ne contenir que des électrons violerait la localité.Votre problème est que vous croyez sans justification que des choses comme les monopôles magnétiques n'existent pas.Mais les trous noirs peuvent transporter des charges magnétiques - ils ne sont de toute façon pas décrits par le modèle standard - et un effondrement générique des aimants produira des trous noirs chargés magnétiquement simplement par GR pur qui est une approximation correcte.
Mais il semble que l'on puisse définir le champ magnétique en termes de champ électrique comme $ B_i = u / c ^ 2 \ epsilon_ {ijk} \ alpha_j E_k $ où $ u_i = u \ alpha_i $ est la vitesse relative au cadre dansdont les charges source ne bougent pas.Par conséquent, il ne semble pas vrai qu'un champ B supplémentaire doive être postulé, il peut être défini en termes de champ E de trame de repos et des vitesses relatives.
Vous connaissiez les champs électriques et magnétiques à 10 ans?
Oui, @ApoorvPotnis - mais ce n'était même pas quelque chose d'exceptionnel.Les champs électriques et magnétiques - leur existence et leurs bases - sont vraiment enseignés à l'école dans mon pays à cet âge.Il y a beaucoup de choses que je savais bien des années avant mes camarades de classe, etc. mais ce n'est pas un exemple.Si vous le souhaitez, je peux trouver des manuels pour les enfants de 10 ans qui traitent de ces domaines, etc.
@LubošMotl Vous écrivez: "Au lieu de cela, il doit s'agir des composantes F0i d'un tenseur antisymétrique à deux indices".Pouvez-vous expliquer comment cela suit?J'ai commencé avec l'hypothèse que les composants du champ électrique doivent être des composants d'un Tensor.Je sais comment ces composants se comportent lors des rotations, c'est ainsi que je conclus que les composants du champ électrique doivent être des composants de type spatial.Mais ici je suis coincé: de quelles hypothèses ai-je besoin pour montrer que le Tensor est de 2e rang et antisymétrique?Dois-je utiliser la propriété de la façon dont les forces se transforment?
Je pense que LubošMotl et @user1247 se sont disputés pour savoir si le champ magnétique est réel.Mais la vraie question de user1247 est de savoir si le monopole magnétique a sa place dans la relativité et la loi de Coulomb a dérivé les équations de Maxwell.Luboš Motl n'a pas répondu à cela dans la première partie de la réponse.Il a répondu à cela dans la dernière partie avec une nouvelle physique, qui dépasse la question initiale de user1247.
Ted Bunn
2011-01-23 05:55:48 UTC
view on stackexchange narkive permalink

La réponse de Lubos Motl est très bonne, mais je pense que cela vaut la peine de dire une ou deux choses supplémentaires.

Vous pouvez considérer le magnétisme comme un simple sous-produit de l'électricité, dans le sens suivant: si vous supposez que la loi de Coulomb est correcte, et que la relativité restreinte est correcte, et que cette charge est un scalaire de Lorentz (donc cette charge et la densité de courant forment un 4-vecteur), alors vous pouvez dériver toutes les équations de Maxwell. (En fait, vous devez probablement également supposer que la théorie est également linéaire, maintenant que j'y pense.) Le manuel de premier cycle de Purcell fonctionne très explicitement d'une manière agréable et agréable, et c'est aussi dans des manuels plus avancés .

Certains livres passent sous silence la nécessité de postuler que la charge est un scalaire. Au moins un manuel - je ne me souviens pas lequel - le souligne et présente un argument convaincant auquel il convient de prêter attention. Une façon de voir que ce n'est pas une condition triviale à imposer est de considérer l'analogie avec la gravité - c'est-à-dire de substituer la masse à la charge et la gravité au champ électrique, et d'essayer d'exécuter le même argument. (Supposons des champs faibles pour que tout puisse être traité comme linéaire si vous le souhaitez.) Il y a des effets «gravitomagnétiques», mais ils ne sont pas liés à la gravité régulière de la même manière que le champ magnétique est lié au champ électrique - c.-à-d. , les analogues gravitationnels des équations de Maxwell semblent différents des équations de Maxwell régulières). Une des raisons est les différences de signe, bien sûr - comme les charges se repoussent dans un cas et attirent dans l'autre. Mais une raison plus importante est que la source de gravité n'est pas un scalaire: sa densité ne fait pas partie d'un 4-vecteur, mais plutôt d'un tenseur de rang 2.

Mais à un niveau plus philosophique (ou peut-être sémantique), je ne passerais pas de ce fait à la conclusion que le magnétisme est "simplement" un sous-produit de l'électricité. À tout le moins, un tel langage ne semble pas utile pour comprendre la théorie ou pour l'utiliser! Par exemple, comprendre comment une onde électromagnétique peut se propager d'une galaxie lointaine vers votre œil est beaucoup plus facile et plus naturel si vous la regardez du point de vue "habituel".

Merci Ted. Donc, si, comme vous le dites, vous pouvez dériver toutes les équations de Maxwell comme un sous-produit de l'électricité, il semble s'ensuivre trivialement que l'on peut écrire les équations de Maxwell sans référence à un champ B. (Tout comme je peux écrire, comme dans l'exercice que j'ai décrit, les forces dues à un courant _I_ sans référence à un champ B). C'est ma question à laquelle Lubos semble refuser de vouloir répondre. Je comprends que cela ne change pas la physique et que ce n'est peut-être pas la manière la plus parcimonieuse d'exprimer l'électromagnétisme - je suis simplement intéressé si cela peut et a été fait.
Oui, ça peut se faire. Avec un effort suffisant, vous pouvez aller plus loin et exprimer toute l'électricité et le magnétisme sans référence à un champ E ou B - tout comme une loi de force * très * étrange et compliquée entre les charges, dans laquelle la force de chaque charge dépend du propriétés de l'autre charge au temps retardé. Le manuel de Griffiths écrit la loi de la force explicitement dans l'un des chapitres ultérieurs. Vous abandonnez beaucoup en faisant cela - la chose la plus importante qui me vient à l'esprit est que je n'ai aucune idée de la façon dont vous essaieriez même de parler de conservation de la dynamique énergétique dans ce langage.
@user1247 Gardez à l'esprit que la parcimonie est * extrêmement * importante.Il y a un nombre infini de théories qui expliquent aussi bien le même phénomène et font les mêmes prédictions.Nous essayons de tout réduire aux fondamentaux.Vous pourriez remplacer tous les champs quantiques par un seul champ plutôt compliqué qui expliquait tout aussi exactement, mais malgré le remplacement de plusieurs champs par un seul, ce serait beaucoup plus complexe.Et nous nous attendons à ce que les lois fondamentales régissant l'univers soient aussi simples que possible (en effet, la «complexité fondamentale» sonne comme un oxymore: D).
asmaier
2011-07-17 01:10:25 UTC
view on stackexchange narkive permalink

Pas une réponse directe à votre question mais toujours une dérivation surprenante des équations de Maxwell:

La preuve de Feynman des équations de Maxwell (FJ Dyson - Phys. Rev. A, 1989 ) montre qu'il est possible de dériver les équations de Maxwell à partir de la deuxième loi de Newton du mouvement et des relations de commutation (sous des limites non relativistes).

Cool! Une copie est disponible ici: http://www.scribd.com/doc/168392117/Freeman-J-Dyson-Feynman%E2%80%99s-proof-of-the-Maxwell-equations-Am-J-Phys- 58-1990-209. Peut être illégal, selon les lois sur le droit d'auteur et l'utilisation équitable dans un pays donné.
Avez-vous lu les commentaires qui suivent?Toutes les équations ne peuvent pas en être dérivées.Bien que ce soit chouette, ce n'est pas complet.
user8817
2012-06-14 16:17:30 UTC
view on stackexchange narkive permalink

Oui, vous pouvez le faire, mais vous devez également utiliser un principe de superposition.

  1. Vous déterminez que la loi de Couloms, $$ \ mathbf F = \ frac {qQ \ mathbf r} {| \ mathbf r | ^ {3}}, $$ est un cas aux limites de la force relativiste, qui agit sur la charge q par le champ d'une charge Q.
  2. Utilisation de la transformation de Lorentz pour la force et pour le rayon-vecteur, $$ \ mathbf F = \ mathbf F '+ \ gamma \ mathbf u \ frac {(\ mathbf F' \ cdot \ mathbf v ')} {c ^ {2}} + \ Gamma \ mathbf u \ frac {(\ mathbf u \ cdot \ mathbf F ')} {c ^ {2}}, $$$$ \ mathbf r' = \ mathbf r + \ Gamma \ mathbf u \ frac {(\ mathbf u \ cdot \ mathbf r)} {c ^ {2}} - \ gamma \ mathbf ut = \ mathbf r + \ Gamma \ mathbf u \ frac {(\ mathbf u \ cdot \ mathbf r)} {c ^ {2 }} (t = 0), $$ où u est la vitesse du système inertiel, v est la vitesse de charge, vous pouvez supposer que par rapport à l'autre système inertiel avec une vitesse relative u la force ressemble à $$ \ mathbf F = q \ mathbf E + \ frac {q} {c} [\ mathbf v \ times \ mathbf B], $$ où $$ \ mathbf E = \ frac {\ gamma Q \ mathbf r} {(r ^ {2} + \ frac {\ gamma ^ {2}} {c ^ {2}} (\ mathbf r \ cdot \ mathbf u) ^ {2}) ^ {\ frac {3} {2}}}, \ quad \ mathbf B = \ frac {1} {c} [\ mathbf u \ times \ mathbf E]. $$ Bien sûr, le champ magnétique est un effet cinématique relativiste, mais une procédure décrite ci-dessus est la transformation cinématographique relativiste de la loi de Coulomb. Donc certaines personnes ont fait une erreur en donnant une réponse négative.
  3. Après cela, en utilisant les théorèmes primaires des analyses vectorielles et la procédure de régularisation, vous pouvez "prendre" pourri et div des expressions E et B ci-dessus. Après cela, vous pouvez gagner les équations de Maxwell. Vous devez utiliser le principe de superposition, lorsque vous passez d'un champ d'une charge à une distribution multi-charge en continu.
+1 pour la vôtre et la réponse WIMP: Lubos et tous les autres ont bien sûr raison, mais cela dépend bien sûr de ce que vous entendez par dériver.Avec les postulats de l'OP traités comme un système axiome formel: bien sûr, vous ne pouvez pas;du point de vue d'un physicien, où vous supposez d'autres choses «raisonnables» comme la linéarité, bien sûr, vous pouvez.
Notez que ce n'est pas réellement une preuve des équations de Maxwell.La relativité restreinte ne s'applique qu'aux charges qui ont (et ont toujours eu) une vitesse constante, mais les équations de Maxwell peuvent en décrire plus que cela.Par exemple en 3. vous démontrez que l'application de div à E conduit à l'une des équations de Maxwell.Cependant, il n'est pas clair que cela fonctionne toujours si la ou les charges ont été accélérées à tout moment et que des champs retardés entrent en jeu.Il s'agit donc d'une hypothèse supplémentaire de votre preuve, qu'elle est toujours valable pour les charges accélérées.
Godfrey Miller
2011-03-08 04:27:13 UTC
view on stackexchange narkive permalink

Je sais que Purcell et d'autres ont utilisé la symétrie de Lorentz comme dispositif pédagogique pour motiver l'introduction des champs magnétiques, mais je ne me souviens pas avoir jamais vu une dérivation axiomatique des équations de Maxwell. Il peut être intéressant de voir précisément quelles hypothèses au-delà de la symétrie de Lorentz et de la loi de Coulomb sont nécessaires pour reconstruire les équations de Maxwell.

Les champs B ne sont pas des champs fictifs

Si vous connaissez les champs magnétiques dans un cadre inertiel, vous pouvez déterminer les champs électriques et magnétiques dans n'importe quel autre cadre via la transformation de Lorentz. Si le champ magnétique disparaît dans une image inertielle donnée, vous pouvez considérer les effets magnétiques dans d'autres images comme fictifs. Cependant, il n'est pas toujours possible de trouver un cadre dans lequel les champs magnétiques disparaissent. Le moyen le plus rapide de voir cela est de noter que E ^ 2 - B ^ 2 c ^ 2 est une quantité invariante de Lorentz ( voir Wikipedia). Si nous trouvons que B ^ 2> E ^ 2 / c ^ 2 à un point d'espace-temps donné dans un référentiel inertiel donné, il s'ensuit que B ^ 2> 0 à ce point dans tous les référentiels inertiels. En fait, vous pourriez commencer dans un cadre où le champ électrique disparaît mais pas le champ magnétique; les champs électriques observés dans d'autres images pourraient alors être considérés comme fictifs.

En général, ni le champ électrique ni le champ magnétique ne peuvent être amenés à disparaître sous un boost de Lorentz. Pour voir cela rapidement, notez que le produit scalaire du vecteur champ E avec le vecteur champ B à un point d'espace-temps donné est une quantité invariante de Lorentz ( voir Wikipedia). Si ce produit scalaire est différent de zéro à un point d'espace-temps donné dans un cadre inertiel donné, les vecteurs de champ électrique et magnétique seront tous deux différents de zéro à ce point d'espace-temps dans tous les cadres inertiels.

Comme l'a souligné Einstein, vous pouvez comprendre le mouvement d'une particule chargée en vous référant au champ électrique dans le cadre de repos de cette particule. Cependant, si vous avez plusieurs particules avec des vitesses différentes, vous devez suivre le champ électrique dans le cadre de repos instantané de chaque particule. Puisque les boosts de Lorentz mélangent le champ E avec le champ B, le seul moyen de garder une trace du champ E dans le cadre de repos de chacune de vos particules en termes de quantités locales dans un référentiel inertiel est en se référant au champ E et au champ B

Localité

Même si c'est possible, il n'est pas clair pour moi qu'il serait souhaitable d'utiliser la loi de Coulomb comme axiome dans la théorie électromagnétique. Les équations de Maxwell expliquent le mouvement des particules en se référant aux degrés de liberté locaux, les champs. La loi de Coulomb, par contre, est une forme d'action à distance, et est manifestement non locale.

Il est certainement possible de réécrire les champs E et B en termes d'intégrales sur la densité de charge et la densité de courant (je ne peux pas poster un autre lien, donc google "les équations de Jefimenko"), puis d'utiliser ces expressions pour interpréter les forces électromagnétiques comme une forme d'action retardée à distance. Cependant, pour obtenir ces expressions, il faut des hypothèses sur les conditions aux limites sur les champs E et B. Nous pouvons toujours obtenir une autre solution valide des équations de Maxwell en changeant simplement les conditions aux limites sur les champs, ce qui démontre que les champs ont une existence indépendante et ne sont pas de simples variables comptables pour simplifier une interaction non locale plus fondamentale.

Monopoles

Comme on l'écrit habituellement, les équations de Maxwell ne contiennent pas de termes correspondant à la charge magnétique, mais il serait cohérent d'ajouter de tels termes. En fait, Dirac a montré que la quantification de la charge électrique pouvait être due à l'existence de monopôles magnétiques (je ne peux pas poster un autre lien, donc google "condition de quantification de dirac monopôle magnétique"). Les équations de Maxwell ne nous disent pas si des monopoles magnétiques existent ou pourraient exister, mais la quantification de la charge électrique pourrait être la preuve que des monopoles magnétiques existent quelque part dans l'univers.

Dans ma réponse, j'ai lié un article de Hans de Vries où il a fait ce que dit user1247, et vous pouvez vérifier sa validité. Dans motionmoutain ch 18 - Motion in GR, on trouve aussi que GravitoElectric est un champ fondamental et que le GravitoMagnetic est un effet relativiste par la même raison. Le mouvement l'induit. Pour moi, c'est une force et non un «champ» (il n'y a pas de champ de Coriolis, mais une force). Comment la particule peut-elle faire cela? "plusieurs particules avec des vitesses différentes, vous devez garder une trace du champ électrique dans le cadre de repos instantané de chaque particule" et "à l'avance"?
Art Brown
2012-06-14 20:58:57 UTC
view on stackexchange narkive permalink

Vous ne pouvez pas. B n'est pas seulement un effet secondaire relativiste de E . Jackson, Electrodynamics , Section 12.2 a une belle discussion, dans laquelle il réfute les «preuves» données dans certains textes de premier cycle.

"La confusion vient principalement du fait que les propriétés de transformation de Lorentz de la force sont telles qu'un terme de force de type magnétique apparaît lorsque la force dans un référentiel inertiel est exprimée en termes de la force dans un autre cadre. Il est tentant de donner à ce terme de force supplémentaire une existence indépendante et d'identifier ainsi le champ magnétique comme une entité distincte. Mais une telle étape est injustifiée sans hypothèses supplémentaires. "

Jackson poursuit en présentant un contre-exemple explicite, basé sur un potentiel scalaire de Lorentz. Ce champ ressemble à l'électrostatique (ou même à la gravitation newtonienne!) Dans la limite non relativiste. Il a également "une force apparente de type magnétique. Mais il n'y a pas d'entité indépendante B ." Donc dans cette "théorie" B n'est en effet qu'un effet relativiste, mais cette théorie ne s'applique pas à la Nature.

12.2?Cinématique des produits de désintégration d'une particule instable?Ou est-ce que je lis mal _Electrodynamics_ par le mauvais Jackson?
@Ruslan, mon ref est Jackson, Classical Electrodynamics, 2e éd.Le titre de la section 12.2 est "Sur la question de l'obtention du champ magnétique, de la force magnétique et des équations de Maxwell à partir de la loi de Coulomb et de la relativité spéciale."
Le titre du chapitre 12 est "Dynamique des particules relativistes et des champs électromagnétiques".
Hmm, essayait la 1ère édition.Mais ce n'est pas non plus présent en 3e.Et je n'ai pas réussi à trouver le 2e.Quoi qu'il en soit, il y a un suivi [article] (http://scitation.aip.org/content/aapt/journal/ajp/54/7/10.1119/1.14521) par Kobe, qui affirme que l'hypothèse manquante pour dériver les équations de Maxwell de Coulombla loi est que la charge est un scalaire conservé.Quoi qu'il en soit, j'aimerais quand même lire le contre-exemple de Jackson que vous citez, donc si vous avez un lien vers la 2ème édition téléchargeable, je vous serais très reconnaissant si vous l'avez posté.
Cette réponse est utile, mais c'est un résumé inexact de ce que dit Jackson.J'ai recherché le passage pertinent de la 2e édition.Il ne dit pas que toutes ces preuves sont fausses, mais seulement qu'il faut faire preuve de prudence pour énoncer certaines hypothèses cachées.Il donne plusieurs références à des traitements qui, selon lui, expliquent correctement les hypothèses: D. H.Frisch et L. Wilets, Am.J. Phys.24, 574 (1956).J. R. Tessman, Am.J. Phys.34, 1048 (1966).M. Schwartz, Principes d'électrodynamique, McGraw-Hill, New York (1972), ch.3.
@BenCrowell, après avoir étudié votre commentaire, je pense toujours avoir résumé Jackson avec précision aux fins de cette question.Je ne conteste certainement pas qu'il y a de bons développements là-bas;en particulier, je connais et j'aime Schwartz, mon texte de premier cycle.Cependant, la question ne demandait pas de références à de bons traitements.Maintenant, il se peut que je manque certaines implications par inadvertance de mon texte: n'hésitez pas à vous améliorer comme bon vous semble.
JxB
2011-02-08 14:03:21 UTC
view on stackexchange narkive permalink

Avec la loi de Coulomb et la relativité restreinte, vous pouvez dériver la loi d'Ampère, qui vous donne la magnétostatique. Ce qui manque à l'électrodynamique, c'est le courant de déplacement ($ \ frac {1} {c ^ 2} \ frac {\ partial E} {\ partial t} $), qui est une source de champ magnétique provenant d'un champ électrique variant dans le temps, et non un résultat du mouvement de la charge électrique.

La relativité n'a que deux postulats:

  1. Les lois de la physique sont les mêmes dans tous les référentiels inertiels
  2. Tous les observateurs inertiels mesurent la même vitesse pour la lumière dans le vide.

La relativité, en elle-même, n'exige pas que les champs électriques (ou le potentiel électrique d'ailleurs) doivent se déplacer à la vitesse de la lumière. Pour dériver les équations de Maxwell, vous avez besoin d'un postulat supplémentaire, fourni par l'équation d'onde (pour le potentiel électrique) dans la section 4 de la référence dans la réponse de Helder. Sans ce postulat supplémentaire (que les changements de potentiel électrique se propagent à la vitesse de la lumière), vous ne pouvez pas dériver le courant de déplacement de la seule loi et relativité de Coulomb.

mais Light IS champ électromagnétique, et vice-versa
@Helder Eh bien, si vous supposez que la relativité restreinte dérive les équations de Maxwell de la manière décrite ici, vous ne pouvez évidemment pas faire référence à la vitesse de la lumière au départ car ce qu'est la lumière n'est pas définie tant que vous n'avez pas terminé votre dérivation.Ce que vous * pouvez * faire, cependant, et devez en fait faire, c'est supposer qu'il existe une vitesse $ v $ qui semble être la même dans tous les cadres de référence.(C'est fondamentalement ce qui distingue la mécanique galiléenne de SR.) Autrement dit, vous formulez SR en utilisant cette vitesse $ v $ (au lieu de la vitesse de la lumière $ c $) et dérivez les équations de Maxwell comme indiqué dans ...
... les autres messages ici.Mais à un moment donné, vous * devrez * faire l'hypothèse que $ v $ est précisément la vitesse à laquelle les perturbations du champ électromagnétique se propagent.Donc JxB a tout à fait raison.
Helder Velez
2011-01-26 17:05:51 UTC
view on stackexchange narkive permalink

par Hans de Vries (*):

La dérivation la plus simple et la plus complète du magnétisme comme effet secondaire relativiste de l'électro-statistique

Il n'utilise que le champ électrostactique et la non-simultanéité pour obtenir le champ magnétique. Il l'explique mieux que Purcell.

Le champ magnétique est un effet secondaire du mouvement dans le champ électrique.

(*) Hans de Vries a un livre en ligne très intéressant (pas encore terminé) dans son site, et il propose une autre perle, sans rapport avec ce post, mais je me sens obligé de partager: La contraction de Lorentz est un effet réel et pas seulement «un effet référentiel» comme on est tenté de le croire.

@JxB Je ne peux pas commenter votre réponse et citant "Pour dériver les équations de Maxwell, vous avez besoin d'un postulat supplémentaire, et qui est fourni par l'équation d'onde (pour le potentiel électrique) dans la section 4 de la référence dans la réponse de Helder. Sans ce postulat supplémentaire ( que les changements de potentiel électrique se propagent à la vitesse de la lumière), vous ne pouvez pas dériver le courant de déplacement de la loi et de la relativité de Coulomb uniquement. "
@JxB suite du commentaire précédent (problème avec la touche Entrée par rapport à ShiftEnter, désolé) ** champ électrique = lumière ** On ne peut pas dissocier le champ électrique de la lumière. "À la vitesse c" ici http://en.wikipedia.org/wiki/Electric_fieldelectr 28 fois ici: http://en.wikipedia.org/wiki/Photon_polarizationexplorez le Radiation2D.exe à partir d'ici http://www-xfel.spring8.or.jp/ Je n'ai aucun doute que le champ électrique, et la gravité, se propage à la vitesse c.
@JxB citant "résultant d'un champ électrique variant dans le temps, et non un résultat du mouvement de la charge électrique." Le mouvement est un concept relatif et un champ électrique «variant dans le temps» est toujours le résultat de charges en mouvement.
pour garder les références à gheter: description de la méthode Radiation2D.exe: "NOUVELLE MÉTHODE MATHÉMATIQUE POUR LE CHAMP DE RAYONNEMENT DE CHARGE DE MOUVEMENT" par T. Shintakehttp: //accelconf.web.cern.ch/accelconf/e02/PAPERS/WEPRI038.pdf
WIMP
2011-01-23 17:11:46 UTC
view on stackexchange narkive permalink

Non, vous ne pouvez pas. Pour plusieurs raisons. Premièrement, si vous avez E, pour obtenir le champ B, vous avez besoin d'hypothèses supplémentaires sur la structure de la théorie, c'est-à-dire plus en détail le tenseur du champ, voir la réponse ci-dessus de Lubos. Mais en plus de cela, même si vous aviez la solution pour une charge ponctuelle, pour obtenir les équations de Maxwell, vous devez en savoir plus qu'une seule solution. Par exemple, qu'ils sont linéaires, de second ordre et quel est le groupe de symétrie. Et si vous avez ajouté cela, vous pouvez de toute façon dériver les équations de Maxwell à partir de ces hypothèses sans même commencer par le champ de Coulomb.

Je dois admettre que certaines hypothèses supplémentaires sont nécessaires.
+1 pour la vôtre et la réponse de PhysiXxx: Lubos et tous les autres ont bien sûr tout à fait raison, mais cela dépend bien sûr de ce que vous entendez par dériver.Avec les postulats de l'OP traités comme un système axiome formel: bien sûr, vous ne pouvez pas;du point de vue d'un physicien, où vous supposez d'autres choses «raisonnables» comme la linéarité, bien sûr, vous pouvez.
John Nygate
2014-08-12 14:20:41 UTC
view on stackexchange narkive permalink

Oui. Voir Principes de l'électrodynamique par Melvin Schwartz. Il dérive toute l'électrodynamique, y compris les équations de Maxwell, de la loi de Coulomb et de la relativité spéciale.

Les réponses constituées uniquement d'un lien ou d'une recommandation de lecture externe sont explicitement de mauvaises réponses.
Il a presque réussi, mais à un moment où il est difficile de combler le fossé qu'il a assorti avec Dieu.À la page 127, il a dit: "En appréciant cela (le tenseur antisymétrique est économique), Dieu a naturellement choisi le tenseur antisymétrique comme son moyen d'expression."Notez que j'adore ce livre.C'est très concis.Je l'ai étudié après celui de Griffith.
Andrew Steane
2018-11-20 19:01:48 UTC
view on stackexchange narkive permalink

La réponse de Luboš Motl est d'une certaine aide, en ce qu'elle montre comment apporter les types d'idées que la relativité offre, mais néanmoins elle s'ouvre sur sa conclusion générale, et cette conclusion est fausse. C'est faux en grande partie pour les raisons brièvement indiquées dans la réponse de WIMP.

La question est importante et il est important de trouver la bonne réponse. La question est:

Les équations de Maxwell peuvent-elles être dérivées en utilisant uniquement la loi de Coulomb et la relativité spéciale?

La réponse est: non, car de nombreuses autres théories de terrain respectant la Relativité Spéciale peuvent être inventées, de manière à reproduire la loi de Coulomb dans le cadre inertiel d'une charge ponctuelle donnée.

Cependant, ce que l’on peut dire, c’est que l’électromagnétisme classique (c’est-à-dire l’équation de Maxwell et l’équation de la force de Lorentz, ou toute formulation équivalente à celle-ci, telle qu’une formulation lagrangienne) fait partie des théories des champs les plus simples qui respectent la Relativité Spéciale et incluent la loi de Coulomb. La définition de «plus simple» ici est certes imprécise.

La principale raison pour laquelle vous ne pouvez pas dériver Maxwell de 'Coulomb + S.R.' c'est que vous ne sauriez pas s'il faut inclure les effets d'accélération dans la relation entre les potentiels et les charges.

Je vais maintenant «lever le couvercle» un peu sur la physique théorique ici. Un très bon (pas le seul) moyen mathématique de s'assurer que tout élément de physique respecte la Relativité Spéciale (S.R.) est de se limiter aux expressions tensorielles dans tout ce que vous proposez et écrivez. Ici, `` tensoriel '' inclut des tenseurs de rang zéro, c'est-à-dire des scalaires, mais pas n'importe quel ancien scalaire: ce seraient des scalaires invariants de Lorentz. Il comprend également 4 vecteurs et des tenseurs de deuxième rang et de rang supérieur. Lorsque vous prenez des dérivées, vous utilisez l'opérateur de gradient covariant $ \ partial_a $ , puis vous disposez d'un kit d'outils pour construire des équations différentielles qui respectent S.R.

La théorie des champs 'la plus simple' pourrait donc être telle que les particules peuvent avoir une propriété scalaire invariante de Lorentz appelée charge $ q $ , et la force sur une charge la particule est indépendante de la 4-vitesse $ u ^ a $ de la particule. Le problème est que vous découvrez rapidement que dans une telle théorie, la force sur une particule ne peut pas changer la vitesse d'une particule sans changer également sa masse. En explorant plus loin, vous essayez de permettre à la 4 forces $ f ^ a $ de dépendre de la 4 vitesses via une équation linéaire simple impliquant un champ scalaire $ \ phi $ , comme $ f ^ a = q \ phi u ^ q $ (?). Toujours pas bon (la masse change à nouveau). Vous êtes donc amené à essayer un tenseur de second rang $ F ^ {ab} $ pour le champ, car c'est la chose la plus simple, autre qu'un scalaire, qui peut prenez un $ u ^ a $ à 4 vecteurs en entrée et rendez une force à 4 vecteurs:

$ f ^ a = q F ^ {a \ mu} u_ \ mu $

Maintenant, c'est ok: la force préserve la masse tant que $ F ^ {ab} $ est antisymétrique. Bien! Un tenseur antisymétrique est le type le plus simple de tenseur de second rang. Ensuite, nous voulons une équation différentielle pour ce champ: essayez la chose la plus simple, qui est de prendre la divergence, et vous êtes bien sur la voie des équations de Maxwell. Si nous introduisons maintenant la loi de Coulomb (et c'est là qu'elle entre en jeu), alors vous êtes assuré d'obtenir deux des équations de Maxwell si vous limitez le terme source dans votre équation différentielle à un seul terme proportionnel à la densité de charge et à la vitesse 4. . La loi de Coulomb ne vous dit pas elle-même de ne pas ajouter d'autres termes liés à l'accélération 4.

Par cette approche, nous n'arrivons pas inexorablement aux équations de Maxwell, mais on constate qu'elles sont sans doute les plus simples qui incluent la propriété de conservation de la charge et qui permettent une force de préservation de masse (en langage technique, un pur force).

Parmi les autres théories des champs que l'on rencontre, il y en a une qui ressemble beaucoup à Maxwell mais qui comprend des monopôles magnétiques. Cela se produit très naturellement, dans le traitement théorique, et est certainement une possibilité sérieuse pour le fonctionnement réel du monde physique. C'est un peu moins simple en ce que l'on perd la belle propriété d'écrire le tenseur de champ comme une 4-curl d'un champ à 4 vecteurs (le 4-potentiel), et la théorie ne respecte plus la symétrie sous inversion spatiale (parité). Voir le livre de Jackson sur l'électromagnétisme pour une discussion. S'il y a en fait des monopôles magnétiques, comme le suggèrent de nombreuses versions de la théorie quantique des champs, alors l'énigme est de savoir pourquoi les monopoles électriques sont tellement plus abondants que les monopôles magnétiques.

Cependant, je voudrais souligner que ce problème de monopole magnétique est loin d'être la seule raison pour laquelle les équations de Maxwell ne sont pas entièrement dérivables de la loi de Coulomb et de S.R. Les autres raisons incluent qu'on peut facilement imaginer que les équations de champ impliquent des dérivées d'ordre supérieur du mouvement de la particule; S.R. à lui seul ne peut pas vous dire qu'ils ne le font pas. En commençant par une approche lagrangienne, on peut introduire d'autres contraintes, telles que l'invariance conduisant à des lois de conservation, puis l'électromagnétisme est assez étroitement, mais pas encore totalement, contraint. Fondamentalement, ce que S.R. peut vous dire, c'est qu'un champ qui fournit une force indépendante de la vitesse d'un corps ne peut pas être toute l'histoire de la physique. Un tel champ (comme le champ électrique) doit être en partenariat avec d'autres effets qui dépendent de la vitesse d'un corps.

Riad
2018-12-29 19:40:14 UTC
view on stackexchange narkive permalink

Il y a peu d'articles qui montrent que l'équation de conservation / continuité pour la charge électrique est suffisante pour dériver l'ensemble des équations de Maxwell. Voir cette référence et des citations par exemple; https://pdfs.semanticscholar.org/3251/31eadb62c8fdfdaaad7b21a308992ff3a4d2.pdf '' Comment obtenir la forme covariante des équations de Maxwell à partir de l'équation de continuité ... Par conséquent, un processus circulaire semble inévitable en électromagnétisme: ρ et J impliquent E et B qui, à leur tour, impliquent de nouveaux ρ1 et J1, et ainsi de suite. En raison de cette caractéristique circulaire, il n’est pas clair si E et B (satisfaisant les équations de Maxwell) sont une conséquence de ρ et J (satisfaisant l’équation de continuité) ou vice versa. Selon l'arbitre, il semble être une question de goût de dire lequel est une conséquence de l'autre. En d'autres termes: à partir du commentaire de l'arbitre, nous pourrions conclure que le lien entre les sources et les champs est un peu comme le problème de l'œuf et de la poule: qui était le premier? ''.

En partant également de la loi de Coulomb pour l'électricité statique et en tenant compte du fait que l'action ne peut pas voyager plus vite que la lumière, l'utilisation de l'intégrale retardée produit l'ensemble complet des équations de Maxwell. https://en.wikipedia.org/wiki/Li%C3%A9nard%E2%80%93Wiechert_potential.

Ainsi, le retard lui-même donne naissance à une force qui est normale au mouvement (vitesse), proportionnelle à celui-ci, et décroissant comme le carré inverse de la distance - qui est le champ magnétique par définition. Il donne également lieu à une force à deux composants - un champ électrique et un champ magnétique proportionnel à l'accélération, qui décroît uniquement comme l'inverse de la distance (et non l'inverse au carré) et c'est le rayonnement par définition. On peut donc en déduire que le magnétisme et le rayonnement sont des phénomènes émergents causés par la finitude de la vitesse de propagation des forces mises en jeu.

Certaines réponses ont indiqué un lien avec le gravito-magnétisme et la relativité.Je pense que cela vient du fait que la loi de gravitation de Newton peut être traitée de la même manière que la loi de Coulomb, donnant lieu à un ensemble d'équations similaires aux équations de Maxwell.Ce sont les équations gravito-magnétiques et sont en fait également dérivables de la relativité générale pour les champs faibles. https://en.wikipedia.org/wiki/Gravitoelectromagnetism

Carlos Beltran
2019-02-22 11:48:47 UTC
view on stackexchange narkive permalink

Si je comprends bien votre idée, vous demandez s'il est possible de récupérer toutes les équations de Maxwell en utilisant uniquement les transformations de Lorentz et en utilisant l'existence d'un champ électrique.La réponse est non.Voici un exemple heuristique: si vous avez un fil circulaire unidimensionnel avec un courant variable $ I (t) $ , il n'y a pas de transformation de Lorentz pour produire le champ magnétique de cesystème partant uniquement d'un champ électrique, car la charge électrique du fil circulaire se déplace de manière non inertielle.

Je pense autrement.Pour chacun des segments infinitésimaux du fil, pour un court intervalle de temps infinitésimal delta t, dans le référentiel inertiel co-mobile dont la vitesse est la même que les électrons dans le fil à cet instant, on peut parler de transformation de Lorentz menant à Maxwelléquations.De plus, le potentiel 4 ne dépend que du courant instantané uniquement et non de ses dérivés, nous pouvons donc le faire.


Ce Q&R a été automatiquement traduit de la langue anglaise.Le contenu original est disponible sur stackexchange, que nous remercions pour la licence cc by-sa 2.0 sous laquelle il est distribué.
Loading...